Đến nội dung

Stranger411 nội dung

Có 85 mục bởi Stranger411 (Tìm giới hạn từ 29-04-2020)



Sắp theo                Sắp xếp  

#346443 Các bài toán về số Ferma - Các bài giảng của LEUNG Tat-Wing

Đã gửi bởi Stranger411 on 13-08-2012 - 13:46 trong Tài liệu, chuyên đề, phương pháp về Số học

Mới tìm được trong đống bài giảng của ông LEUNG Tat-Wing 1 bài viết rất hay về Số Ferma.

Công nhận mấy giáo sư người Tàu rất đáng khâm phục. Mời các bạn tham khảo nó :D

File gửi kèm




#346426 Tìm các số nguyên dương $a,b,c$ sao cho $\frac{a^{2}+b^{2...

Đã gửi bởi Stranger411 on 13-08-2012 - 12:03 trong Số học

ý m` là không có cách của bạn đâu,nếu 2a+b chia hết cho p thì làm quoái j` mà vô lí chứ,cũng giống như a mũ 2 + b mũ 2 có ước nguyên tố dạng thì 4p+3 thì a,b cùng chia hết cho SNT đó,làm j` có chuyện vô lí ở đâu,khi sử dụng kí hiêu lengdre ta đã ngầm hiểu tử số của nó không chia hết cho mẫu r` :lol:

Trước khi nói cái gì thì nên coi lại kiến thức của mình một tí đi nhá ;)

Định nghĩa về kí hiệu Lengdre:
Cho số nguyên tố $p$ và số nguyên $a$. Khi đó, ta có:
+$\left( {\frac{{ a}}{p}} \right) = 1$ nếu $a \not \vdots p$ và $a$ là số chính phương $mod(p)$
+$\left( {\frac{{ a}}{p}} \right) = -1$ nếu $a \not \vdots p$ và $a$ không là số chính phương $mod(p)$
+$\left( {\frac{{ a}}{p}} \right) = 0$ nếu $a \vdots p$

Trờ lại bài toán:
Vì vậy nếu $\left( {\frac{{ - 3}}{p}} \right) = 1$ thì hoàn toàn vô lí vì ta chọn $p \equiv 2(\bmod 3)$



#346412 $(p+1)(p+2)...(2p-1) \equiv (p-1)! (\mod p^3)$

Đã gửi bởi Stranger411 on 13-08-2012 - 11:02 trong Số học

Chứng minh rằng với số nguyên tố $p>3$ thì
$$(p+1)(p+2)...(2p-1) \equiv (p-1)! (\mod p^3)$$

Một số mở rộng cho Wolstenholme’s Theorem.

File gửi kèm

  • File gửi kèm  aa7144.pdf   189.33K   311 Số lần tải



#346392 Tìm các số nguyên dương $a,b,c$ sao cho $\frac{a^{2}+b^{2...

Đã gửi bởi Stranger411 on 13-08-2012 - 09:49 trong Số học

muốn có -3 là thặng dư toàn phương của p thì 1 trong 2 số 2a+b hoặc b không chia hết cho p,nên theo nguyenta a,b,c chia hết cho p là hợp lí

Gì nữa đây bạn :P
$\left( {\frac{{ - 3}}{p}} \right) = 1$ là kí hiệu Lengdre ;) Chớ có phải thăng dư toàn phuơng gì đâu ;))
Cách em Nguyenta98 là lùi vô hạn. Cách mình là dùng các định lí về thăng dư bậc 2 thôi ;)



#346389 Hội những người độc thân thích chém gió !

Đã gửi bởi Stranger411 on 13-08-2012 - 09:42 trong Góc giao lưu

Duyệt cho anh =))

Ố ố :-ss
Mình là con gái mà ;)
Anh nào đâu ;))



#346020 Tìm các số nguyên dương $a,b,c$ sao cho $\frac{a^{2}+b^{2...

Đã gửi bởi Stranger411 on 11-08-2012 - 23:47 trong Số học

He he áp dụng cái bổ đề anh Tường nói thì bài này làm ngon
Giải như sau:
Bổ đề: $p \in \mathbb{P}, p \equiv 2 \pmod{3}, a^2+3b^2 \vdots p \Leftrightarrow p|a,b$

Em Tạ giải kinh quá :P
Chắc thằng Tường nó chả bao giờ theo kịp đâu :P

Cách khác:
Đẳng thức được viết lại như sau $(a+b+c)^2=(3k+2)(ab+bc+ca)$
Chọn số nguyên tố $p$ sao cho $\left\{ \begin{gathered}
{p^{2a - 1}}|3k + 2 \\
{p^{2a}}|3k + 2 \\
\end{gathered} \right.$
$ \Rightarrow \left\{ \begin{gathered}
{p^a}|a + b + c \\
p|ab + bc + ca \\
\end{gathered} \right.$
$c \equiv - a - b(\bmod p) \Rightarrow p|a^2 + ab + b^2 \Rightarrow p|{(2a + b)^2} + 3{b^2}$
$ \Rightarrow \left( {\frac{{ - 3}}{p}} \right) = 1$. Và điều này vô lí vì $p \equiv 2(\bmod 3)$.
Vậy không tồn tại $a,b,c$ thỏa mãn bài toán. $\blacksquare$



#345861 Ảnh thành viên

Đã gửi bởi Stranger411 on 11-08-2012 - 14:58 trong Góc giao lưu

Hình đã gửi
Up cái này phát!
Ông quả nhìn to quá!

Mấy ông ơi !!
cái bảng đó bây h ai giữ thế ;p
Bữa nào bán đấu giá trên Vmf đi :D



#345850 $\varphi (5^{m}-1)=5^{n}-1$

Đã gửi bởi Stranger411 on 11-08-2012 - 14:38 trong Số học

Ặc, ý em nó là nếu $a_i-1=0$ thì cái $p_i$ có tồn tại đâu mà số chính phương $mod(p)$ nữa? :D

Ko ko, em nhầm hàng rồi :|
mình ko xét cái $\varphi (5^{m}-1)$ đâu
chỉ cần xét ${5^m} - 1 = {2^a}{p_1}^{{a_1}} \ldots {p_k}^{{a_k}}$.
Từ cái này ta chứng minh 5 là số chính phương $mod(p)$ là ok rồi ;)



#345845 $\varphi (5^{m}-1)=5^{n}-1$

Đã gửi bởi Stranger411 on 11-08-2012 - 14:15 trong Số học

He he, thú thực vs anh là em làm y như anh đó, nhưng cả bài của anh và em đều thiếu một cái quan trọng đó là mũ của $p_i$ có thể bằng $0$ anh hiểu ko (hay $a_i-1=0$ đó) nên chưa làm gì được ở đoạn số chính phương $mod(p)$ đâu :D cho nên bài này vẫn cần phải nghĩ thêm


Very beautiful !! A problem in quadratic residue ;)

Bổ đề: Cho các số nguyên $m,n$ và $a>1$. Ta có: $\gcd \left( {{a^m} - 1,{a^n} - 1} \right) = {a^{\gcd \left( {m,n} \right)}} - 1$

Lời giải bài toán:
Vì ${2^3}|{5^m} - 1 \Rightarrow m = 2k + 1 \Rightarrow {p_i}|5.{\left( {{5^k}} \right)^2} - 1$
Vậy 5 là số chính phương (mod $p_i$) nên ${p_i} \equiv - 1(\bmod 5)$
$$ \Rightarrow \left\{ \begin{gathered}
{\left( { - 1} \right)^k} = 1 \\
{\left( { - 2} \right)^{k + 1}} \equiv 1(\bmod 5) \\
\end{gathered} \right.
\Rightarrow \left\{ \begin{gathered}
k \equiv 0(\bmod 2) \\
k \equiv 3(\bmod 4) \\
\end{gathered} \right.$$
Mâu thuẫn,
Vậy $gcd(m,n)>1$.

Thực ra ko phải nghĩ gì cho nhiều em à :P
Từ đoạn này trở xuống mình xét ${5^m} - 1$ nên chả có liên quan gì đến $a_i -1=0$ cả :-j
Với lại $a_i -1=0$ ko quan trọng khi chứng minh $a=2$ ở phần trên :D

Và từ đẳng thức:
\[{5^n} - 1 = \varphi \left( {{5^m} - 1} \right) = {2^{a - 1}}{p_1}^{{a_1} - 1}...{p_k}^{{a_k} - 1}\prod\limits_{i = 1}^k {\left( {{p_i} - 1} \right)} \]
cho ta ${p_i} \not \equiv 1(\bmod 5)$ cũng chẳng liên quan gì đến $a_i -1=0$ em à :P



#345840 $\varphi (5^{m}-1)=5^{n}-1$

Đã gửi bởi Stranger411 on 11-08-2012 - 13:36 trong Số học

giả sử m,n là các số nguyên dương sao cho $\varphi (5^{m}-1)=5^{n}-1$.cmr UCLN của m và n lớn hơn 1

Very beautiful !! A problem in quadratic residue ;)

Bổ đề: Cho các số nguyên $m,n$ và $a>1$. Ta có: $\gcd \left( {{a^m} - 1,{a^n} - 1} \right) = {a^{\gcd \left( {m,n} \right)}} - 1$

Lời giải bài toán:
Giả sử $gcd(m,n)=1$
Ta xét phân tích cơ sở: ${5^m} - 1 = {2^a}{p_1}^{{a_1}} \ldots {p_k}^{{a_k}}$ với ${p_i} > 2$
Vậy nên:
\[{5^n} - 1 = \varphi \left( {{5^m} - 1} \right) = {2^{a - 1}}{p_1}^{{a_1} - 1}...{p_k}^{{a_k} - 1}\prod\limits_{i = 1}^k {\left( {{p_i} - 1} \right)} \]
Vì ${2^a}|{5^n} - 1$. Kết hợp với bổ đề, ta có: $\gcd \left( {{5^m} - {{1,5}^n} - 1} \right) = 5 - 1 = 4$
Vậy nên $a=2$
Vì ${2^3}|{5^m} - 1 \Rightarrow m = 2k + 1 \Rightarrow {p_i}|5.{\left( {{5^k}} \right)^2} - 1$
Vậy 5 là số chính phương (mod $p_i$) nên ${p_i} \equiv - 1(\bmod 5)$
$$ \Rightarrow \left\{ \begin{gathered}
{\left( { - 1} \right)^k} = 1 \\
{\left( { - 2} \right)^{k + 1}} \equiv 1(\bmod 5) \\
\end{gathered} \right.
\Rightarrow \left\{ \begin{gathered}
k \equiv 0(\bmod 2) \\
k \equiv 3(\bmod 4) \\
\end{gathered} \right.$$
Mâu thuẫn,
Vậy $gcd(m,n)>1$.



#345818 $\frac{{{a^2} - 2}}{{2...

Đã gửi bởi Stranger411 on 11-08-2012 - 12:21 trong Số học

Tìm $a,b \in {\mathbb{Z}^ + }$ sao cho $\frac{{{a^2} - 2}}{{2{b^2} + 3}} \in \mathbb{Z}$



#344948 CMR$\left( {\frac{{p - 1}}{2...

Đã gửi bởi Stranger411 on 08-08-2012 - 23:38 trong Số học

Bài toán trên còn 1 cách phát biểu khác như sau:
Hình đã gửi



#343961 $A = \frac{{{a^2} + {b^2} + {c^2...

Đã gửi bởi Stranger411 on 06-08-2012 - 12:19 trong Số học

Tìm các số nguyên dương a,b,c đề $A = \frac{{{a^2} + {b^2} + {c^2}}}{{abc}} \in {N^*}$

Bài này vô số nghiệm.

Bước 1: Ta cần chứng minh: $A=1$ hoặc $A=3$.

Bước 2: Ta tìm $a,b,c$
Với $A=3$, nghiệm của phương trình $a^2+b^2+c^2=3abc$ là
\[\left\{ \begin{gathered}
z = 1 \\
x = {u_n} \\
y = {u_{n + 1}} \\
\end{gathered} \right.\]
Với
\[\left\{ \begin{gathered}
{u_0} = 1,{u_1} = 1\\
{u_{n + 1}} = 3{u_n} - {u_{n - 1}}\\
\end{gathered} \right.\]

Với $A=1$. Đặt $a=3x,b=3y,c=3z$, ta được phương trình trên.
Dẫn đến một bộ nghiệm tương tự ;)



#343333 Đếm số cách lát các quân đôminô

Đã gửi bởi Stranger411 on 04-08-2012 - 13:57 trong Tổ hợp và rời rạc

Cho một hình chữ nhật có kích thước $2 \times n$ được đánh số thứ tự từ trái sang phải là $1,2,, \cdot,n$ ở hàng thứ nhất và $n+1,n+2, \cdot,2n$ ở hàng thứ 2.

Lát chúng bằng các quân đôminô $1\times2$ sao cho
1) Phủ kín hình chữ nhật và ko có 2 quân nào chồng lền nhau.
2) Vơi $n$ lẻ, ta được phép bổ sung thêm 1 quân đôminô "đặc biệt" sao cho có thể phủ kín ô $n$ và $n+1$
Đếm số cách lát thỏa mãn đề bài.



ps: Thực chất đây là bài toán tập hợp liên quan đến các phần tử tốt.



#343075 MOSP 2001 by Cecil Rousseseau

Đã gửi bởi Stranger411 on 03-08-2012 - 13:15 trong Tổ hợp và rời rạc

Problem: $a_n$ kí hiệu là số tập con không rỗng của $S$ thỏa mãn rằng:
(i) $S\subseteq${$1$, $2$, $...$, $n$};
(ii) tất cả các phần tử của $S$ đều cung tính chẵn, lẻ.
(iii) mỗi phân tử $k\in{S}$ thỏa mãn $k\geq2|S|$.
Tìm công thức tường minh cho $a_n$

Bài này lâu rồi, sử dụng phép chia nhóm là được :)

Ta có: $ a_{2m-1}= 2(F_{m+1}-1) $ và $ a_{2m}= F_{m+3}-2 $
với $m\ge1$ và $F_{m}$ là số Fibonacci thứ $m$.


Lời giải:
Đặt $T_{n}=\{S\in\{1,2,\cdots,n\}\}$ thỏa $(ii)$ và $(iii)$
Chia $ T_{n+4} $ thành 3 tập con:

Phần 1: $A_{n+4}=\{S\in T_{n+4}\ ;\ 1,2\notin S,\ \forall k\in S, k\geq 2|S|+2\}$
Xây dựng $ f\ :\ \mathcal{P}(\{1,2,\cdots,n+2\})\rightarrow\mathcal{P}(\{1,2,\cdots,n+4\}) $ thỏa:
$$f(\{x_{1},x_{2},\cdots,x_{k}\}) =\{x_{1}+2,x_{2}+2,\cdots,x_{k}+2\}$$
Ta được: $ f(T_{n+2}) = A_{n+4} $ nên $ |A_{n+4}|=|T_{n+2}|$

Phần 2: $ B_{n+4}=\{S\in T_{n+4}\ ;\ 1,2\notin S,\ \exists k\in S, k < 2|S|+2\} $
Tương tự, ta được:

$f(\phi) =\{3\}$
$f\left( {\left\{ {{x_1},{x_2},...,{x_k}} \right\}} \right) = \left\{ {{x_1} + 4,{x_2} + 4,...,{x_k} + 4,2k} \right\}$
nếu các phần tử cùng chẳn.
$f\left( {\left\{ {{x_1},{x_2},...,{x_k}} \right\}} \right) = \left\{ {{x_1} + 4,{x_2} + 4,...,{x_k} + 4,2k+1} \right\}$
nếu các phần tử cùng lẽ.
Suy ra: $|B_{n+4}|=|T_{n}|+1 $

Phần 3: $ C_{n+4}=\{ S\in T_{n+4}\ ;\ 1\in S\ \mathrm{or}\ 2\in S\} $
Tương tự, ta có: $ |T_{n+4}|=|T_{n+2}|+|T_{n}|+2 $

Từ đó, ta chứng minh được: $ a_{2m-1}= 2(F_{m+1}-1) $ và $ a_{2m}= F_{m+3}-2 $



#343069 bài toán về tập tốt

Đã gửi bởi Stranger411 on 03-08-2012 - 12:53 trong Tổ hợp và rời rạc

Bài toán đã có ở đây :D

Bulgaria TST 2003



#343066 Chứng minh giá trị của $k$ thuộc 1 tập xác định

Đã gửi bởi Stranger411 on 03-08-2012 - 12:48 trong Số học

Chứng minh rằng nếu tồn tại $ a;b\in\mathbb{Z}^{+} $ sao cho $ \frac{(a-b)^{2}+m}{pab-q}= k (m\in\mathbb{N}^{*}) $ với $p,q$ được xác định như trên thì giá trị của $k$ thuộc 1 tập xác định.



#343028 có tất cả bao nhiêu số tự nhiên n<m sao cho m l n(2n+1)(5n+2)

Đã gửi bởi Stranger411 on 03-08-2012 - 11:27 trong Số học

cho m=20072008 ,hỏi có tất cả bao nhiêu số tự nhiên n<m sao cho m l n(2n+1)(5n+2)

\[{\text{m}} = {\text{2}}{007^{2008}} = {3^{4016}}{223^{2008}}\]
Ta có: $m|n\left( {2n + 1} \right)\left( {5n + 2} \right)$
$ \Rightarrow m|10n\left( {10n + 5} \right)\left( {10n + 4} \right)$ $(1)$

Đặt $10n=x$, ta được: $m|x\left( {x + 5} \right)\left( {x + 4} \right)$

Đặt ${q_1} = {3^{4016}},{q_2} = {223^{2008}}$. Vì $\gcd \left( {{q_1},{q_2}} \right) = 1$

Nên \[{\text{(1)}} \Leftrightarrow \left\{ \begin{gathered}
x\left( {x + 5} \right)\left( {x + 4} \right) \equiv 0(\bmod {q_1}) \\
x\left( {x + 5} \right)\left( {x + 4} \right) \equiv 0(\bmod {q_2}) \\
\end{gathered} \right.\]
Mà $x \equiv 0(\bmod 10)$

Nên $x$ là nghiệm của hệ:
\[\left\{ \begin{gathered}
x \equiv {r_1}(\bmod {q_1}) \\
x \equiv {r_2}(\bmod {q_2}) \\
x \equiv 0(\bmod 10) \\
\end{gathered} \right.\]
với ${r_1},{r_2} \in \left\{ {0; - 4; - 5} \right\}$

Theo định lí Thặng Dư Trung Hoa, hệ có 1 nghiệm $(\bmod 10{q_1}{q_2})$
với mỗi cặp ${r_1},{r_2}$ chỉ tồn tại 1 nghiệm x.

Có tất cả $3^2$ cách chọn ${r_1},{r_2}$ nên có 9 số $x$ thỏa.
Suy ra có 9 số $n$ thỏa mãn bài toán.



#340870 Chứng minh: $a = {10^k}$

Đã gửi bởi Stranger411 on 27-07-2012 - 20:43 trong Số học

Cho n=1.Ta có S(a+1)=2
Suy ra a+1 có dạng:\[a + 1 = 2 \times {10^k}\] hoặc \[a + 1 = {10^k} + {10^h}(k > h)\]

Đến đây có thể dùng 2 tính chất quan trọng của $S(n)$ để giải bài toán.
là $S(m)+S(n) \ge S(m+n)$ và $S(m)S(n) \ge S(mn)$



#340310 Chứng minh: $a = {10^k}$

Đã gửi bởi Stranger411 on 26-07-2012 - 01:01 trong Số học

Cho số nguyên dương $a$ thỏa mãn $S\left( {{a^n} + n} \right) = 1 + S\left( n \right)$ với mọi số tự nhiên $n$ lớn tùy ý.
Chứng minh $a$ là một lũy thừa của $10$.

- Kvant -




#340308 $S\left( {{a_n}} \right)$ không chia...

Đã gửi bởi Stranger411 on 26-07-2012 - 00:51 trong Tổ hợp và rời rạc

Cho 1 dãy số nguyên $\left( {{a_n}} \right)$ phân biệt thỏa mãn ${{a_n} \leqslant 4999n}$ ${\forall n \geqslant 1}$.
Chứng minh có vô hạn số $n$ sao cho $S\left( {{a_n}} \right) $ không chia hết cho $5$



#339956 $\sum\limits_{k=1}^{p-1}{({...

Đã gửi bởi Stranger411 on 25-07-2012 - 10:29 trong Tổ hợp và rời rạc

Ở đầu bài toán có đk là p>3
mình sẽ tiếp tục lời giải của bạn để chứng minh chia hết cho $p^3$
ta có $\sum_{k=1}^{p-1}\binom{p}{k}^2=\binom{2p}{p}-2$
mà theo định lí Wolstenholme ta có $\binom{2p}{p} \equiv 2 (mod p^3)$
phát biểu Định lí http://chuyentoanpbc...2/06/trang1.jpg

Em năm nay 12 mà chả biết mấy cái này :mellow:
Em ko bit đánh giá thế nào nên phải dựa vào cách chứng minh của định lí Willson nên nó hơi dài 1 tí :mellow:

Ta chứng minh:
$\sum\limits_{k=1}^{p-1}{{{\left( C_{p}^{k} \right)}^{2}}\equiv 1(\bmod \,\,{{p}^{3}})}$ (1)

$\Leftrightarrow \sum\limits_{k=1}^{p-1}{{{\left( \frac{(p-1)!}{k!(p-k)!} \right)}^{2}}\equiv 0(\bmod \,\,\,p)}$ (2)

Với mỗi $k\in \text{ }\!\!\{\!\!\text{ 1}\text{,2},...,\text{ p-1}\}$ đặt ${{a}_{k}}=\frac{(p-1)!}{k!(p-k)!}$
$ \Leftrightarrow k!.{{a}_{k}}=(p-1)(p-2)...(p-k+1) $
$ \Leftrightarrow k.{{a}_{k}}\equiv {{(-1)}^{k-1}}(\bmod \,\,\,p) $ (3)

Xét ${{b}_{k}}=\frac{(p-1)!}{k}$, $\forall k\in \left\{ 1,2,...,p-1 \right\}$.

Theo Định lý Wison ta có $k{{b}_{k}}\equiv (-1)(\bmod \,\,\,p)$. (4)

Từ (3) và (4) ta có :
${{a}_{k}}\equiv {{(-1)}^{k}}{{b}_{k}}(\bmod \,\,p)$ (5)

Do $p$ là số nguyên tố và $k\in \left\{ 1,2,...,p-1 \right\}$ nên tồn tại duy nhất $j\in \left\{ 1,2,...,p-1 \right\}$ sao cho:
$(kj)\equiv 1(\bmod \,\,\,p)$$\Rightarrow $${{(kj)}^{2}}\equiv 1(\bmod \,\,\,p)$.

Khi đó:
$$\sum\limits_{k=1}^{p-1}{{{({{b}_{k}})}^{2}}}=\sum\limits_{k=1}^{p-1}{\left( {{({{b}_{k}})}^{2}}.1 \right)}\equiv \sum\limits_{k=1}^{p-1}{\left( {{({{b}_{k}})}^{2}}.{{(kj)}^{2}} \right)}\equiv \left( (p-1)! \right)\sum\limits_{j=1}^{p-1}{{{j}^{2}}(\bmod \,\,\,p)}$$

$$\sum\limits_{j=1}^{p-1}{{{j}^{2}}=\frac{p(p-1)(2p-1)}{6}\equiv 0(\bmod \,\,\,p)}$$
nên $\sum\limits_{k=1}^{p-1}{{{({{b}_{k}})}^{2}}}\equiv 0(\bmod \,\,\,p)$ (6)

Từ (5) và (6) suy ra $\sum\limits_{k=1}^{p-1}{{{({{a}_{k}})}^{2}}}\equiv 0(\bmod \,\,\,p)$ hay (2) đúng.



#339925 $\sum\limits_{k=1}^{p-1}{({...

Đã gửi bởi Stranger411 on 25-07-2012 - 09:53 trong Tổ hợp và rời rạc

Có vẻ như bài này chỉ chứng minh chia hết cho $p^2$ thôi, thử với $p=3$ có vẻ không đúng.

Rõ ràng là :
$$ \sum\limits_{k=1}^{p-1}(C_p^{k})^2 \vdots p^2$$

Anh gì đó ơi, bài này có thể quy về chứng minh:
\[\sum\limits_{k=1}^{p-1}{{{\left( C_{p}^{k} \right)}^{2}}\equiv 1(\bmod \,\,{{p}^{3}})}\]
$$\Leftrightarrow \sum\limits_{k=1}^{p-1}{{{\left( \frac{(p-1)!}{k!(p-k)!} \right)}^{2}}\equiv 0(\bmod \,\,\,p)}$$


Với mỗi $k\in \text{ }\!\!\{\!\!\text{ 1}\text{,2},...,\text{ p-1}\}$ đặt ${{a}_{k}}=\frac{(p-1)!}{k!(p-k)!}$
$$ \Leftrightarrow k!.{{a}_{k}}=(p-1)(p-2)...(p-k+1) $$
$$ \Leftrightarrow k.{{a}_{k}}\equiv {{(-1)}^{k-1}}(\bmod \,\,\,p) $$

Đến đây, dùng định lí Willson thôi anh ạ :lol:
Mà $p>3$ mà anh :icon6:



#339875 $\sum\limits_{k=1}^{p-1}{({...

Đã gửi bởi Stranger411 on 25-07-2012 - 07:42 trong Tổ hợp và rời rạc

Cho số nguyên tố $p>3$ và tập hợp $M=\left\{ 1,2,...,p \right\}$. Với mỗi số nguyên $k$ thỏa mãn $1\le k\le p$ ta đặt : ${{E}_{k}}=\left\{ A\subset M:|A|=k \right\}$ và ${{x}_{k}}=\sum\limits_{A\in {{E}_{k}}}{\left( \min A+\max A \right)}$. Chứng minh rằng:
$$\sum\limits_{k=1}^{p-1}{({{x}_{k}}C_{p}^{k})\equiv0(\bmod \,\,{{p}^{3}})}$$



#314274 $$\dfrac{a^3+b^3+c^3+3abc}{(a+b+c)(ab+bc+ca)}+\dfrac{abc}...

Đã gửi bởi Stranger411 on 04-05-2012 - 13:03 trong Bất đẳng thức - Cực trị

Bài toán :
Cho $a, b, c \ge 0$ . Chứng minh rằng :
$$\dfrac{a^3+b^3+c^3+3abc}{(a+b+c)(ab+bc+ca)}+\dfrac{abc}{a^2b+b^2c+c^2a}\ge 1$$
Nguồn : ML

Bài toán của bạn được ghép từ 3 bổ đề sau:
1)${{a}^{3}}+{{b}^{3}}+{{c}^{3}}+3abc\ge \sum{ab\left( a+b \right)}$


2)$\left( a+b+c \right)\left( ab+bc+ca \right)\frac{8}{9}\le \left( a+b \right)\left( b+c \right)\left( c+a \right)$

3)$\frac{a}{b+c}+\frac{b}{c+a}+\frac{c}{a+b}+\frac{3abc}{2\left( {{a}^{2}}b+{{b}^{2}}c+{{c}^{2}}a \right)}\ge 2$